Difference between revisions of "2021 Fall AMC 12B Problems/Problem 23"

(Solution 1)
(Solution 3)
Line 18: Line 18:
 
~MT
 
~MT
  
== Solution 3 ==
+
==Solution 3 (casework bash)==
We define an outcome as <math>\left( a_1 ,\cdots, a_5 \right)</math> with <math>1 \leq a_1 < a_2 < a_3 < a_4 < a_5 \leq 30</math>.
 
  
We denote by <math>\Omega</math> the sample space. Hence. <math>| \Omega | = \binom{30}{5}</math>.
+
We will proceed with some casework.
 +
Let <math>n</math> be the number of sets of consecutive numbers in the subset.
 +
Note that the maximum possible value of <math>n</math> is <math>4.</math>
 +
Case <math>1</math>: <math>n = 4</math>
 +
There is only one way to arrange the distribution of the number of elements in each block here. There are <math>\dbinom{26}{1} = 26</math> ways to arrange where that block of <math>5</math> numbers will go, so a total of <math>1 \cdot 26 = 26</math> ways for this case.
 +
Case <math>2</math>: <math>n = 3</math>
 +
There are <math>4</math> ways to arrange the distribution of the number of elements in each block here. (4-1, 3-2, 2-3, 1-4). There are <math>\dbinom{26}{2}</math> ways to arrange where those two blocks of numbers go among the other numbers such that no two of those blocks are adjacent to each other. Total for this case: <math>4 \cdot 325 = 1300.</math>
 +
Case <math>3</math>: <math>n = 2</math>
 +
By Stars and Bars, there are <math>\dbinom{4}{2} = 6</math> ways to arrange the distribution of the number of elements in each block here. There are <math>\dbinom{26}{3}</math> ways to arrange where those three blocks of numbers go among the other numbers such that no two of those blocks are adjacent to each other. Total for this case: <math>6 \cdot 2600 = 15600.</math>
 +
Case <math>4</math>: <math>n = 1</math>
 +
By Stars and Bars, there are <math>\dbinom{4}{3} = 4</math> ways to arrange the distribution of the number of elements in each block here. There are <math>\dbinom{26}{4}</math> ways to arrange where those four blocks of numbers go among the other numbers such that no two of those blocks are adjacent to each other. Total for this case: <math>4 \cdot 14950 = 59800.</math>
 +
Since the last case <math>n=0</math> doesn't contribute to the expected value sum, we can ignore it.
 +
Using the expected value formula, our desired value is <cmath>\frac{4 \cdot 26 + 3 \cdot 1300 + 2 \cdot 15600 + 1 \cdot 59800}{\tbinom{30}{5}} = \frac{95004}{142506} = \boxed{\frac{2}{3}}.</cmath>
  
<math>\textbf{Case 1}</math>: There is only 1 pair of consecutive integers.
+
-fidgetboss_4000
 
 
<math>\textbf{Case 1.1}</math>: <math>\left( a_1 , a_2 \right)</math> is the single pair of consecutive integers.
 
 
 
We denote by <math>E_{11}</math> the collection of outcomes satisfying this condition.
 
Hence, <math>| E_{11} |</math> is the number of outcomes satisfying
 
<cmath>
 
\[
 
\left\{
 
\begin{array}{l}
 
a_1 \geq 1 \\
 
a_3 \geq a_1 + 3 \\
 
a_4 \geq a_3 + 2 \\
 
a_5 \geq a_4 + 2 \\
 
a_5 \leq 30 \\
 
a_1, a_3, a_4, a_5 \in \Bbb N
 
\end{array}
 
\right..
 
\]
 
</cmath>
 
 
 
Denote <math>b_1 = a_1 - 1</math>, <math>b_2 = a_3 - a_1 - 3</math>, <math>b_3 = a_4 - a_3 - 2</math>, <math>b_4 = a_5 - a_4 - 2</math>, <math>b_5 = 30 - a_5</math>.
 
Hence,
 
<math>| E_{11} |</math> is the number of outcomes satisfying
 
<cmath>
 
\[
 
\left\{
 
\begin{array}{l}
 
b_1 + b_2 + b_3 + b_4 + b_5 = 22 \\
 
b_1, b_2 , b_3, b_4, b_5 \mbox{ are non-negative integers }
 
\end{array}
 
\right..
 
\]
 
</cmath>
 
 
 
Therefore, <math>| E_{11} | = \binom{22 + 5 - 1}{5 - 1} = \binom{26}{4}</math>.
 
 
 
<math>\textbf{Case 1.2}</math>: <math>\left( a_2 , a_3 \right)</math> is the single pair of consecutive integers.
 
 
 
We denote by <math>E_{12}</math> the collection of outcomes satisfying this condition.
 
Hence, <math>| E_{12} |</math> is the number of outcomes satisfying
 
<cmath>
 
\[
 
\left\{
 
\begin{array}{l}
 
a_1 \geq 1 \\
 
a_2 \geq a_1 + 2 \\
 
a_4 \geq a_2 + 3 \\
 
a_5 \geq a_4 + 2 \\
 
a_5 \leq 30 \\
 
a_1, a_2, a_4, a_5 \in \Bbb N
 
\end{array}
 
\right..
 
\]
 
</cmath>
 
 
 
Similar to our analysis for Case 1.1,  <math>| E_{12} | = \binom{22 + 5 - 1}{5 - 1} = \binom{26}{4}</math>.
 
 
 
<math>\textbf{Case 1.3}</math>: <math>\left( a_3 , a_4 \right)</math> is the single pair of consecutive integers.
 
 
 
We denote by <math>E_{13}</math> the collection of outcomes satisfying this condition.
 
Hence, <math>| E_{13} |</math> is the number of outcomes satisfying
 
<cmath>
 
\[
 
\left\{
 
\begin{array}{l}
 
a_1 \geq 1 \\
 
a_2 \geq a_1 + 2 \\
 
a_3 \geq a_2 + 2 \\
 
a_5 \geq a_3 + 3 \\
 
a_5 \leq 30 \\
 
a_1, a_2, a_3, a_5 \in \Bbb N
 
\end{array}
 
\right..
 
\]
 
</cmath>
 
 
 
Similar to our analysis for Case 1.1,  <math>| E_{13} | = \binom{22 + 5 - 1}{5 - 1} = \binom{26}{4}</math>.
 
 
 
<math>\textbf{Case 1.4}</math>: <math>\left( a_4 , a_5 \right)</math> is the single pair of consecutive integers.
 
 
 
We denote by <math>E_{14}</math> the collection of outcomes satisfying this condition.
 
Hence, <math>| E_{14} |</math> is the number of outcomes satisfying
 
<cmath>
 
\[
 
\left\{
 
\begin{array}{l}
 
a_1 \geq 1 \\
 
a_2 \geq a_1 + 2 \\
 
a_3 \geq a_2 + 2 \\
 
a_4 \geq a_3 + 2 \\
 
a_4 \leq 29 \\
 
a_1, a_2, a_3, a_4 \in \Bbb N
 
\end{array}
 
\right..
 
\]
 
</cmath>
 
 
 
Similar to our analysis for Case 1.1,  <math>| E_{14} | = \binom{22 + 5 - 1}{5 - 1} = \binom{26}{4}</math>.
 
 
 
<math>\textbf{Case 2}</math>: There are 2 pairs of consecutive integers.
 
 
 
<math>\textbf{Case 2.1}</math>: <math>\left( a_1 , a_2 \right)</math> and <math>\left( a_2 , a_3 \right)</math> are two pairs of consecutive integers.
 
 
 
We denote by <math>E_{21}</math> the collection of outcomes satisfying this condition.
 
Hence, <math>| E_{21} |</math> is the number of outcomes satisfying
 
<cmath>
 
\[
 
\left\{
 
\begin{array}{l}
 
a_1 \geq 1 \\
 
a_4 \geq a_1 + 4 \\
 
a_5 \geq a_4 + 2 \\
 
a_5 \leq 30 \\
 
a_1, a_4, a_5 \in \Bbb N
 
\end{array}
 
\right..
 
\]
 
</cmath>
 
 
 
Similar to our analysis for Case 1.1,  <math>| E_{21} | = \binom{23 + 4 - 1}{4 - 1} = \binom{26}{3}</math>.
 
 
 
<math>\textbf{Case 2.2}</math>: <math>\left( a_1 , a_2 \right)</math> and <math>\left( a_3 , a_4 \right)</math> are two pairs of consecutive integers.
 
 
 
We denote by <math>E_{22}</math> the collection of outcomes satisfying this condition.
 
Hence, <math>| E_{22} |</math> is the number of outcomes satisfying
 
<cmath>
 
\[
 
\left\{
 
\begin{array}{l}
 
a_1 \geq 1 \\
 
a_3 \geq a_1 + 3 \\
 
a_5 \geq a_3 + 3 \\
 
a_5 \leq 30 \\
 
a_1, a_3, a_5 \in \Bbb N
 
\end{array}
 
\right..
 
\]
 
</cmath>
 
 
 
Similar to our analysis for Case 1.1,  <math>| E_{22} | = \binom{23 + 4 - 1}{4 - 1} = \binom{26}{3}</math>.
 
 
 
<math>\textbf{Case 2.3}</math>: <math>\left( a_1 , a_2 \right)</math> and <math>\left( a_4 , a_5 \right)</math> are two pairs of consecutive integers.
 
 
 
We denote by <math>E_{23}</math> the collection of outcomes satisfying this condition.
 
Hence, <math>| E_{23} |</math> is the number of outcomes satisfying
 
<cmath>
 
\[
 
\left\{
 
\begin{array}{l}
 
a_1 \geq 1 \\
 
a_3 \geq a_1 + 3 \\
 
a_4 \geq a_3 + 2 \\
 
a_4 \leq 29 \\
 
a_1, a_3, a_4 \in \Bbb N
 
\end{array}
 
\right..
 
\]
 
</cmath>
 
 
 
Similar to our analysis for Case 1.1,  <math>| E_{23} | = \binom{23 + 4 - 1}{4 - 1} = \binom{26}{3}</math>.
 
 
 
<math>\textbf{Case 2.4}</math>: <math>\left( a_2 , a_3 \right)</math> and <math>\left( a_3 , a_4 \right)</math> are two pairs of consecutive integers.
 
 
 
We denote by <math>E_{24}</math> the collection of outcomes satisfying this condition.
 
Hence, <math>| E_{24} |</math> is the number of outcomes satisfying
 
<cmath>
 
\[
 
\left\{
 
\begin{array}{l}
 
a_1 \geq 1 \\
 
a_2 \geq a_1 + 2 \\
 
a_5 \geq a_2 + 4 \\
 
a_5 \leq 30 \\
 
a_1, a_2, a_5 \in \Bbb N
 
\end{array}
 
\right..
 
\]
 
</cmath>
 
 
 
Similar to our analysis for Case 1.1,  <math>| E_{24} | = \binom{23 + 4 - 1}{4 - 1} = \binom{26}{3}</math>.
 
 
 
<math>\textbf{Case 2.5}</math>: <math>\left( a_2 , a_3 \right)</math> and <math>\left( a_4 , a_5 \right)</math> are two pairs of consecutive integers.
 
 
 
We denote by <math>E_{25}</math> the collection of outcomes satisfying this condition.
 
Hence, <math>| E_{25} |</math> is the number of outcomes satisfying
 
<cmath>
 
\[
 
\left\{
 
\begin{array}{l}
 
a_1 \geq 1 \\
 
a_2 \geq a_1 + 2 \\
 
a_4 \geq a_2 + 3 \\
 
a_4 \leq 29 \\
 
a_1, a_2, a_4 \in \Bbb N
 
\end{array}
 
\right..
 
\]
 
</cmath>
 
 
 
Similar to our analysis for Case 1.1,  <math>| E_{25} | = \binom{23 + 4 - 1}{4 - 1} = \binom{26}{3}</math>.
 
 
 
<math>\textbf{Case 2.6}</math>: <math>\left( a_3 , a_4 \right)</math> and <math>\left( a_4 , a_5 \right)</math> are two pairs of consecutive integers.
 
 
 
We denote by <math>E_{26}</math> the collection of outcomes satisfying this condition.
 
Hence, <math>| E_{26} |</math> is the number of outcomes satisfying
 
<cmath>
 
\[
 
\left\{
 
\begin{array}{l}
 
a_1 \geq 1 \\
 
a_2 \geq a_1 + 2 \\
 
a_3 \geq a_2 + 2 \\
 
a_3 \leq 28 \\
 
a_1, a_2, a_3 \in \Bbb N
 
\end{array}
 
\right..
 
\]
 
</cmath>
 
 
 
Similar to our analysis for Case 1.1,  <math>| E_{26} | = \binom{23 + 4 - 1}{4 - 1} = \binom{26}{3}</math>.
 
 
 
<math>\textbf{Case 3}</math>: There are 3 pairs of consecutive integers.
 
 
 
<math>\textbf{Case 3.1}</math>: <math>\left( a_1 , a_2 \right)</math>, <math>\left( a_2 , a_3 \right)</math> and <math>\left( a_3 , a_4 \right)</math> are three pairs of consecutive integers.
 
 
 
We denote by <math>E_{31}</math> the collection of outcomes satisfying this condition.
 
Hence, <math>| E_{31} |</math> is the number of outcomes satisfying
 
<cmath>
 
\[
 
\left\{
 
\begin{array}{l}
 
a_1 \geq 1 \\
 
a_5 \geq a_1 + 5 \\
 
a_5 \leq 30 \\
 
a_1, a_5 \in \Bbb N
 
\end{array}
 
\right..
 
\]
 
</cmath>
 
 
 
Similar to our analysis for Case 1.1,  <math>| E_{31} | = \binom{24 + 3 - 1}{3 - 1} = \binom{26}{2}</math>.
 
 
 
<math>\textbf{Case 3.2}</math>: <math>\left( a_1 , a_2 \right)</math>, <math>\left( a_2 , a_3 \right)</math> and <math>\left( a_4 , a_5 \right)</math> are three pairs of consecutive integers.
 
 
 
We denote by <math>E_{32}</math> the collection of outcomes satisfying this condition.
 
Hence, <math>| E_{32} |</math> is the number of outcomes satisfying
 
<cmath>
 
\[
 
\left\{
 
\begin{array}{l}
 
a_1 \geq 1 \\
 
a_4 \geq a_1 + 4 \\
 
a_4 \leq 29 \\
 
a_1, a_4 \in \Bbb N
 
\end{array}
 
\right..
 
\]
 
</cmath>
 
 
 
Similar to our analysis for Case 1.1,  <math>| E_{32} | = \binom{24 + 3 - 1}{3 - 1} = \binom{26}{2}</math>.
 
 
 
<math>\textbf{Case 3.3}</math>: <math>\left( a_1 , a_2 \right)</math>, <math>\left( a_3 , a_4 \right)</math> and <math>\left( a_4 , a_5 \right)</math> are three pairs of consecutive integers.
 
 
 
We denote by <math>E_{33}</math> the collection of outcomes satisfying this condition.
 
Hence, <math>| E_{33} |</math> is the number of outcomes satisfying
 
<cmath>
 
\[
 
\left\{
 
\begin{array}{l}
 
a_1 \geq 1 \\
 
a_3 \geq a_1 + 3 \\
 
a_3 \leq 28 \\
 
a_1, a_3 \in \Bbb N
 
\end{array}
 
\right..
 
\]
 
</cmath>
 
 
 
Similar to our analysis for Case 1.1,  <math>| E_{33} | = \binom{24 + 3 - 1}{3 - 1} = \binom{26}{2}</math>.
 
 
 
<math>\textbf{Case 3.4}</math>: <math>\left( a_2 , a_3 \right)</math>, <math>\left( a_3 , a_4 \right)</math> and <math>\left( a_4 , a_5 \right)</math> are three pairs of consecutive integers.
 
 
 
We denote by <math>E_{34}</math> the collection of outcomes satisfying this condition.
 
Hence, <math>| E_{34} |</math> is the number of outcomes satisfying
 
<cmath>
 
\[
 
\left\{
 
\begin{array}{l}
 
a_1 \geq 1 \\
 
a_2 \geq a_1 + 2 \\
 
a_2 \leq 27 \\
 
a_1, a_2 \in \Bbb N
 
\end{array}
 
\right..
 
\]
 
</cmath>
 
 
 
Similar to our analysis for Case 1.1,  <math>| E_{34} | = \binom{24 + 3 - 1}{3 - 1} = \binom{26}{2}</math>.
 
 
 
<math>\textbf{Case 4}</math>: There are 4 pairs of consecutive integers.
 
 
 
In this case, <math>\left( a_1, a_2 , a_3 , a_4 , a_5 \right)</math> are consecutive integers.
 
 
 
We denote by <math>E_4</math> the collection of outcomes satisfying this condition.
 
Hence, <math>| E_4 |</math> is the number of outcomes satisfying
 
<cmath>
 
\[
 
\left\{
 
\begin{array}{l}
 
a_1 \geq 1 \\
 
a_1 \leq 27 \\
 
a_1 \in \Bbb N
 
\end{array}
 
\right..
 
\]
 
</cmath>
 
 
 
Hence, <math>| E_4 | = 26</math>.
 
 
 
Therefore, the average number of pairs of consecutive integers is
 
<cmath>
 
\begin{align*}
 
& \frac{1}{| \Omega|}
 
\left(
 
1 \cdot \sum_{i=1}^4 | E_{1i} |
 
+ 2 \cdot \sum_{i=1}^6 | E_{2i} |
 
+ 3 \cdot \sum_{i=1}^4 | E_{3i} |
 
+ 4 \cdot  | E_4 |
 
\right) \\
 
& = \frac{1}{\binom{30}{5}}
 
\left(
 
4 \binom{26}{4} + 12 \binom{26}{3} + 12 \binom{26}{2} + 4 \cdot 26
 
\right) \\
 
& = \frac{2}{3} .
 
\end{align*}
 
</cmath>
 
 
 
Therefore, the answer is <math>\boxed{\textbf{(A) }\frac{2}{3}}</math>.
 
  
 
==Solution 4 (casework bash)==
 
==Solution 4 (casework bash)==

Revision as of 17:18, 9 June 2022

Problem

What is the average number of pairs of consecutive integers in a randomly selected subset of $5$ distinct integers chosen from the set $\{ 1, 2, 3, …, 30\}$? (For example the set $\{1, 17, 18, 19, 30\}$ has $2$ pairs of consecutive integers.)

$\textbf{(A)}\ \frac{2}{3} \qquad\textbf{(B)}\ \frac{29}{36} \qquad\textbf{(C)}\ \frac{5}{6} \qquad\textbf{(D)}\ \frac{29}{30} \qquad\textbf{(E)}\ 1$

Solution 1

There are $29$ possible pairs of consecutive integers, namely $p_1=\{1,2\},  \cdots, p_{29}=\{29,30\}$. Define a random variable $X_i$, with $X_i=1$, if $p_i$ is part of the 5-element subset, and $0$ otherwise. Then the number of pairs of consecutive integers in a $5$-element selection is given by the sum $X_1+\cdots + X_{29}$. By linearity of expectation, the expected value is equal to the sum of the $\mathbb{E}[X_i]$: \[\mathbb{E}[X_1+\cdots +X_{29}]=\mathbb{E}[X_1]+\cdots + \mathbb{E}[X_{29}]\] To compute $\mathbb{E}[X_i]$, note that $X_i=1$ for a total of $\binom{28}{3}$ out of $\binom{30}{5}$ possible selections. Thus\[\mathbb{E}[X_i]=\frac{\binom{28}{3}}{\binom{30}{5}}= \frac 1{29}\cdot \frac 23, \quad \textrm{which implies} \quad \mathbb{E}[X_1+\cdots +X_{29}]= \boxed{\textbf{(A)}\ \frac{2}{3}}.\] ~kingofpineapplz

Solution 2

Alternatively, using linearity of expectation on the chosen subset: there are ${5 \choose 2}$ pairs of integers. There are 29 pairs of consecutive integers out of ${30 \choose 2}$ possible pairs, thus the probability that any given pair is consecutive is $\frac{29}{{30 \choose 2}}$. By linearity of expectation, there are $\frac{{5 \choose 2}29}{{30 \choose 2}} = \frac{10 \cdot 29}{\frac{30 \cdot 29}{2}} = \boxed{\textbf{(A)}\ \frac{2}{3}}$ such pairs on average.

~MT

Solution 3 (casework bash)

We will proceed with some casework. Let $n$ be the number of sets of consecutive numbers in the subset. Note that the maximum possible value of $n$ is $4.$ Case $1$: $n = 4$ There is only one way to arrange the distribution of the number of elements in each block here. There are $\dbinom{26}{1} = 26$ ways to arrange where that block of $5$ numbers will go, so a total of $1 \cdot 26 = 26$ ways for this case. Case $2$: $n = 3$ There are $4$ ways to arrange the distribution of the number of elements in each block here. (4-1, 3-2, 2-3, 1-4). There are $\dbinom{26}{2}$ ways to arrange where those two blocks of numbers go among the other numbers such that no two of those blocks are adjacent to each other. Total for this case: $4 \cdot 325 = 1300.$ Case $3$: $n = 2$ By Stars and Bars, there are $\dbinom{4}{2} = 6$ ways to arrange the distribution of the number of elements in each block here. There are $\dbinom{26}{3}$ ways to arrange where those three blocks of numbers go among the other numbers such that no two of those blocks are adjacent to each other. Total for this case: $6 \cdot 2600 = 15600.$ Case $4$: $n = 1$ By Stars and Bars, there are $\dbinom{4}{3} = 4$ ways to arrange the distribution of the number of elements in each block here. There are $\dbinom{26}{4}$ ways to arrange where those four blocks of numbers go among the other numbers such that no two of those blocks are adjacent to each other. Total for this case: $4 \cdot 14950 = 59800.$ Since the last case $n=0$ doesn't contribute to the expected value sum, we can ignore it. Using the expected value formula, our desired value is \[\frac{4 \cdot 26 + 3 \cdot 1300 + 2 \cdot 15600 + 1 \cdot 59800}{\tbinom{30}{5}} = \frac{95004}{142506} = \boxed{\frac{2}{3}}.\]

-fidgetboss_4000

Solution 4 (casework bash)

We will proceed with some casework. Let $n$ be the number of sets of consecutive numbers in the subset. Note that the maximum possible value of $n$ is $4.$ Case $1$: $n = 4$ There is only one way to arrange the distribution of the number of elements in each block here. There are $\dbinom{26}{1} = 26$ ways to arrange where that block of $5$ numbers will go, so a total of $1 \cdot 26 = 26$ ways for this case. Case $2$: $n = 3$ There are $4$ ways to arrange the distribution of the number of elements in each block here. (4-1, 3-2, 2-3, 1-4). There are $\dbinom{26}{2}$ ways to arrange where those two blocks of numbers go among the other numbers such that no two of those blocks are adjacent to each other. Total for this case: $4 \cdot 325 = 1300.$ Case $3$: $n = 2$ By Stars and Bars, there are $\dbinom{4}{2} = 6$ ways to arrange the distribution of the number of elements in each block here. There are $\dbinom{26}{3}$ ways to arrange where those three blocks of numbers go among the other numbers such that no two of those blocks are adjacent to each other. Total for this case: $6 \cdot 2600 = 15600.$ Case $4$: $n = 1$ By Stars and Bars, there are $\dbinom{4}{3} = 4$ ways to arrange the distribution of the number of elements in each block here. There are $\dbinom{26}{4}$ ways to arrange where those four blocks of numbers go among the other numbers such that no two of those blocks are adjacent to each other. Total for this case: $4 \cdot 14950 = 59800.$ Since the last case $n=0$ doesn't contribute to the expected value sum, we can ignore it. Using the expected value formula, our desired value is \[\frac{4 \cdot 26 + 3 \cdot 1300 + 2 \cdot 15600 + 1 \cdot 59800}{\tbinom{30}{5}} = \frac{95004}{142506} = \boxed{\frac{2}{3}}.\]

-fidgetboss_4000

~Steven Chen (www.professorchenedu.com)

See Also

2021 Fall AMC 12B (ProblemsAnswer KeyResources)
Preceded by
Problem 22
Followed by
Problem 24
1 2 3 4 5 6 7 8 9 10 11 12 13 14 15 16 17 18 19 20 21 22 23 24 25
All AMC 12 Problems and Solutions

The problems on this page are copyrighted by the Mathematical Association of America's American Mathematics Competitions. AMC logo.png